MCAT Practice Exam #4

Pataasin ang iyong marka sa homework at exams ngayon gamit ang Quizwiz!

Case study design

(not a longitudinal study design) allows researchers the opportunity to conduct in-depth analyses of a few select participants, then emerge with very detailed data

Cross-sectional research

(not a longitudinal study design) gives researchers a variety of participants of different types (e.g., different ages) that they can track at the same time

Many forces influence how people relate to one another, become socialized beings, and learn to develop relationships. Which of the following is NOT one of the agents of socialization? A. Family B. School C. Travel D. Television

. While travel can be highly educational in terms of introducing people to other cultures and ways of life, it is not specifically an agent of socialization; it does not necessarily help people learn how to relate, as one can travel alone and avoid contact with others (choice C is not an agent of socialization and is therefore correct). Family is one of the first and primary sources of socialization from which a person learns how to relate to others --- one's parents, siblings, and extended family (depending on the family structure). Lessons learned from the family often form the basis for many future relationship choices (choice A is an agent of socialization and is therefore wrong). School is another primary source of socialization, as teachers try to educate children about the values of the specific culture and the important skills needed to survive in that culture (choice B is an agent of socialization and is therefore wrong). Television, along with other forms of mass media, provide people with a glimpse of how to relate to the larger world, introducing people to other lifestyles as well as popular trends and ideas (choice D is also an agent of socialization and is therefore wrong).

If the distance of an object from a converging lens is x and the distance of the real image from the lens is also x, what is the focal length of the lens? A. x / 2 B. x C. 2x D. 3x

A. (1/o)+(1/i)=(1/f) (1/x)+(1/x)=(1/f) (2/x)=(1/f) f=x/2

(R)-2-Methylcyclohexanone was placed in a flask containing 0.9 equivalents of LDA at −78°C. The flask was then warmed to 0°C and benzaldehyde was added to the solution. How many stereoisomeric products can be isolated from the mixture? A. 4 B. 3 C. 2 D. 1

A. (R)-2-Methylcyclohexanone contains a single stereocenter. When the ketone is added to LDA, the less substituted carbon is deprotonated generating the kinetic enolate. This does NOT affect the stereocenter on the more substituted carbon. However, there is more ketone than base, and as a result not all of the ketone is deprotonated. Thus when the solution is warmed to 0°C, the kinetic enolate tautomerizes to the more stable thermodynamic enolate. This racemizes the stereocenter of the enolate as it adds to the benzaldehyde. Since both the top and bottom of the π system of the enolate can react with either the top or bottom of the π system of the benzaldehyde, the products formed will each have two stereocenters, and all possible combinations of configurations will be produced. Therefore, four possible stereoisomeric products can be isolated.

H2CO3(aq) --> HCO3-(aq) + H+(aq) What is the effect of adding a small amount of a strong acid to the equilibrium shown in Equation 1? A. The equilibrium shifts to the left. B. The pH shifts to 7. C. The pH decreases dramatically. D. The concentration of HCO3- increases.

A. According to Le Châtelier's principle, adding a small amount of acid (H+) to this solution will shift the equilibrium to the left (choice A). This will result in a decrease in the concentration of acetate ion, making choice D incorrect. In addition, since the solution is buffered, it will resist changes in pH, making choices B and C incorrect.

If the ionizing gas were evacuated from the Geiger counter tube while it was in operation, which of the following would occur? A. The charge on the cylindrical shell would decrease. B. The charge on the cylindrical shell would increase. C. The voltage between the cylindrical shell and the wire would increase. D. The voltage between the cylindrical shell and the wire would decrease.

A. According to the passage, the cylindrical shell and wire act as a capacitor. The gas in between the two therefore acts as a dielectric. Removal of that gas (replacing it with vacuum) reduces the dielectric constant to 1 between the wire and the cylinder, thus reducing capacitance. If capacitance decreases while the high voltage battery remains connected (as it does while the Geiger counter is in operation, thereby maintaining the voltage between the cylindrical shell and the wire, eliminating choices C and D), the charge must also decrease according to Q = CV.

Isolated damage to the left side of the visual cortex would: A. impair vision of the right side of the visual field. B. impair vision of the left side of the visual field. C. impair perception of complex patterns only. D. impair color vision only.

A. According to the passage, the left side of the visual cortex controls the right side of the visual field, and vice versa (choice A is correct and choice B can be eliminated). Perception of complex patterns is a function of V2, which is located in the both hemispheres, and not the left side only (choice C can be eliminated). Choice D can also be eliminated: color vision is related to a variety of processes from the eye to the brain, and is not limited to the left side of the brain only.

In the semantic network model, what determines the strength of a connection between a node and an association? A. How frequently and deeply connections are made B. How closely the node and association are explicitly related C. How many links there are between the node and the association D. How many state-dependent cues have been created between the node and the association

A. According to the semantic network model, the strength of a connection between a node and an association is related to how frequently and how deeply the connections are made (choice A is correct). The explicit relationship between the node and the association is meaningless unless that connection is constantly reinforced (choice B is wrong). Separate links between the node and the association—suggesting a more circuitous route—are indicative of the spreading activation pattern (choice C is wrong). State-dependent cues, in which familiar locales are used to trigger memories, do not determine the strength of the connection between a node and an association unless, once again, they are used with great frequency (choice D is wrong).

Dietary creatine supplements have many effects on skeletal muscle, such as altering levels of myostatin, a protein produced and released by myocytes. Myostatin alters the autocrine function of muscle cells, thus inhibiting myogenesis. Creatine also affects satellite cells, small multipotent cells located between the basement membrane and the sarcolemma of muscle fibers. These cells have virtually no cytoplasm but are able to proliferate and donate additional myonuclei to their parent muscle fiber Creatine supplementation would cause which of the following effects? A. Increased activity of satellite cells and lower serum myostatin levels B. Decreased activity of satellite cells and lower serum myostatin levels C. Increased activity of satellite cells and higher serum myostatin levels D. Decreased activity of satellite cells and higher serum myostatin levels

A. Creatine supplementation is taken by athletes to increase performance. The passage says that satellite cells proliferate and donate additional myonuclei to their parent muscle fiber, which would be a good thing when training (choices B and D are wrong). The passage also says that myostatin inhibits myogenesis, which would hinder muscle building. Creatine therefore likely inhibits myostatin release (choice C is wrong; choice A is correct).

An individual's description of her behavior at work earlier that day is an example of retrieval from: A. episodic memory. B. semantic memory. C. procedural memory. D. olfactory memory.

A. Episodic memory refers to memory of one's personal experiences (choice A is correct). Semantic memory is memory of concepts and information (choice B is wrong), while procedural memory is memory of how to execute motor tasks (choice C is wrong). Olfactory memory is not a term associated with models of long-term memory. In the context of short-term memory, it would refer to a sensory store from which one could recall what a given stimulus smelled like. It would not have relevance to recall of one's behavior (choice D is wrong).

Which one of the following processes does NOT take place in the mitochondrion? A. Fatty acid synthesis B. Krebs cycle C. Electron transport D. Oxidation of pyruvic acid

A. Fatty acid synthesis occurs in the cytosol (choice A is correct). The Krebs cycle, electron transport, and the oxidation of pyruvic acid all happen in the mitochondrion (choices B, C, and D are wrong).

If the plasma glucose falls below normal, which of the following conditions is true regarding pancreatic activity? A. High levels of glucagon will be produced from alpha cells. B. High levels of insulin will be produced from beta cells. C. Pancreatic acinar cells will secrete high levels of somatostatin. D. The pancreas will produce high levels of both insulin and glucagon.

A. Glucagon is secreted by alpha cells in the pancreas and acts to elevate plasma glucose if it falls too low (choice A is correct). Insulin acts to lower blood glucose and will not be secreted in response to low blood glucose (choices B and D are wrong). Somatostatin is secreted in response to raised glucose or other nutrients and acts to repress digestive functions, so it will not be elevated in response to decreased glucose (choice C is wrong).

Like α(1→4)-glycosidase, which of the following enzymes acts to cleave α(1→4) linkages between glucose molecules? A. Glycogen phosphorylase B. Glucose-6-phosphatase C. Phosphoglucomutase D. Pyrophosphatase

A. Glycogen phosphorylase catalyzes the rate limiting step of glycogenolysis—the release of glucose-1-phosphate from glycogen via its action on terminal α(1→4) linkages. This makes choice A true and the correct answer. Glucose-6-phosphatase hydrolyzes glucose-6-phosphate, resulting in the release of a phosphate group and free glucose for export from the cell. Choice B is false and an incorrect answer. Phosphoglucomutase is involved in both glycogenolysis and glycogenesis by interconverting glucose 1-phosphate and glucose 6-phosphate, thus choice C is false and the incorrect answer. Pyrophosphatases are members of the acid hydrolase family of enzymes that cleave diphosphate bonds. Choice D is false and an incorrect answer.

All four Maaza brothers are two meters tall but each has a different weight with the oldest one weighing 900 N. Assuming the apparatus remains standing, what is the approximate net torque on support pole number 2 when the oldest Maaza is on the tightrope? A. 0 N·m B. 450 N·m C. 900 N·m D. 27,000 N·m

A. If the net torque on either support pole were anything but zero, the pole would accelerate rotationally, and the apparatus would not remain standing.

In order to maintain the maximum yield of ammonia when raising the temperature of the reaction, scientists should: A. maintain N2 and H2 at high pressure. B. allow N2 and H2 to expand with the increased kinetic energy. C. increase the volume in the reaction chamber to favor the forward reaction. D. carefully control the temperature to avoid runaway heating by the forward reaction.

A. In order to maximize the yield of the reaction while increasing temperature, the pressure should be kept as high as possible. Maintaining the compression of the gases (choice A) will maintain a high pressure. However, increasing the volume (choices B or C) would decrease the pressure and lower the overall yield. Since the reaction is exothermic, its forward progress might heat the reaction, hindering the formation of products. Controlling the temperature would eliminate this (choice D).

After an individual has diverged from a social group, that group would be considered: I. the individual's out-group. II. the individual's in-group. III. the individual's reference group. A. I only B. II only C. III only D. I and II only

A. Item I is true: an out-group is defined as a social group with which the individual does not identify, which is the same concept as a dissimilar group from which someone would tend to diverge. After an individual diverges from a social group, that group would be considered his or her out-group (choice B can be eliminated). Item II is false: an in-group is defined as a social group with which the individual identifies; an individual might choose to diverge from a group because he or she does not want to be associated with that group or be mistaken for a member. After the person has diverged, that group would not be considered his or her in-group (choice D can be eliminated). Item III is false: reference groups are used as a way for individuals to compare themselves and their cultural tastes to a social group with which they identify. After an individual has diverged from a social group, that group would not be considered his or her reference group (choice C can be eliminated, choice A is correct).

How might a theorist in the tradition of symbolic interactionism explain deviance? A. Deviant behavior is a response to being labeled a deviant by others. B. Deviant behavior results from performing on the back stage when one should be performing on the front stage. C. Deviance is a failure to develop a looking-glass self. D. Deviance is a failure to resolve the dilemma of identity versus role confusion in adolescence.

A. Labeling theory is a fundamental concept of symbolic interactionism. It states that deviance is not inherent in the act itself, but is rather an attribution from others (such as police, family, or clergy; choice A is correct). Back stage and front stage dynamics are an aspect of the dramaturgical approach. If one acts on the back stage when he or she should be performing on the front stage, the audience might experience more than it otherwise would (e.g., more details about a person's motivations or inner thoughts), but this would not necessarily be a deviant act (choice B is wrong). A looking-glass self describes how people shape their self-concepts based on how they think others perceive them (choice C is wrong). The dilemma of identity versus role confusion is one of Erikson's eight developmental stages; it is not an aspect of symbolic interactionism (choice D is wrong).

According to Lawrence Kohlberg, what is the highest level of morality that the average adult tends to achieve? A. She or he will acknowledge a sense of duty to uphold the laws of the land and maintain basic social conventions. B. He or she will strive for social justice—the greatest good for the greatest number. C. He or she will focus on the approval (or disapproval) of others, acting in accordance with the views of his or her community or peer group. D. She or he will act in such a way as to avoid punishment at all costs.

A. Lawrence Kohlberg postulated six stages of moral development, but said that the average person generally does not pass beyond the fourth stage, in which he or she acknowledges a sense of duty to uphold the law and maintain social conventions (choice A is correct). Striving for social justice, wherein one acts for the greatest good for the greatest number, is the fifth stage in Kohlberg's schema and one that he proposed that few people reach (choice B is wrong). A focus on receiving approval from one's peers along with a need to conform is the third stage of development, usually seen in older children and adolescents (choice C is wrong). Acting to avoid punishment is the earliest stage of moral development, typical of very young children (choice D is wrong).

Contraction of cardiac muscle begins just after depolarization and lasts about as long as the action potential. During a contraction: A. Ca2+ enters the cell through voltage-gated ion channels. B. sarcomere length increases. C. polymerized actin becomes depolymerized. D. myosin binds irreversibly to actin

A. Part of the cardiac action potential is the opening of slow voltage-gated calcium channels to create the plateau in depolarization (Phase 2) so choice A is correct. When the voltage-gated calcium channels open, calcium enters the cell and can play a role in the contraction of the cardiac muscle. Sarcomere length decreases during each contraction, and does not increase (choice B is wrong). Actin filaments in contractile tissue are not dynamic as they are in other processes and do not spontaneously depolymerize and repolymerize (choice C is wrong). Myosin binds actin reversibly, not irreversibly (choice D is wrong).

Cells from the pigmented region of a frog gastrula are transplanted to a light gastrula region. The light region surrounding the graft is induced to form a second incomplete tadpole. Cells transplanted from a light region to the same site do not induce formation of a second tadpole. Based on the results of this experiment, the developmental fate of the region surrounding the graft most likely depends on: A. the region from which the graft cells were taken. B. the region to which the graft cells are transplanted. C. the expression of pigment genes. D. the migration of the graft cells throughout the transplant region.

A. Pigmented cells induce development of an incomplete tadpole, while cells from the light region do not when transplanted to the same area. The region from which the cells are taken appears more important for developmental fate than the area they are transplanted to (choice A is correct, and choice B is wrong). Pigmented cells appear to be the key factor, but pigmentation itself is likely to be coincidental and not the cause of the developmental fate of the region (choice A is a better answer than choice C). There is no evidence presented that migration of cells is important (choice D is wrong).

Media representations of Planned Parenthood often center on their abortion practices. Planned Parenthood is responsible for an annual total of 300,000 abortions. However, this accounts for a mere three percent of the services provided. This misrepresentation of Planned Parenthood is an example of: A. institutional discrimination. B. the McDonaldization of society. C. a coercive organization. D. symbolic interactionism.

A. Planned Parenthood experiences the effects of prejudice as a result of practices, such as abortion, that conflict with the beliefs of some groups, and this prejudice, in some cases, contributes to institutional discrimination. Mass media is a powerful institution in the United States, and the media executives with higher socioeconomic status are able to set the agenda for their programs. This can lead to institutional discrimination in situations when the media presents a specific group in a negative light for its own purposes (e.g., the anti-abortion movement; choice A is correct). The McDonaldization of society described when a culture adopts the characteristics of a fast-food restaurant, such as efficiency, calculability, standardization, and control. While aspects of mass media are certainly a good example of McDonaldization, the misrepresentation of Planned Parenthood by the media is not an example of this (choice B is wrong). A coercive organization is one where members have no choice in joining; for example, a prison is an example of a coercive organization. While Planned Parenthood has little choice in what the media portrays about them, as an organization, Planned Parenthood is not a part of the media - these are two separate organizations. Therefore, this is not an example of a coercive organization (choice C is wrong). Symbolic interactionism is a major sociological theory that suggests that people create symbolic meanings about objects, events, and behaviors. For example, a handshake is a symbol in society that conveys many different meanings—and everyone in society pretty much acts toward this symbol in agreed-upon ways. Symbolic interactionism is a micro-level theory of society, which focuses on interactions between individuals. Therefore, even if the interaction between the media and Planned Parenthood neatly conveyed something symbolic, it still could not be an example of symbolic interactionism because these are two social structures, which is a macro-level view of society (choice D is wrong).

Suppose Subject 405 returned home after the experiment and watched a television program about mammalian predators. Before a commercial break, the program's narrator explained that bears secrete kairomones and challenged viewers to think of the primary chemical contained in bear kairomone. During the break subject 405 confidently told her roommate that "no doubt the primary chemical is sulfur!" Assuming that subject 405 has no familiarity with kairomones other than through her experience as a study participant, her assertion exemplifies which of the following? A. The availability heuristic B. The optimistic bias C. The representativeness heuristic D. The central route to persuasion

A. The availability heuristic refers to people's tendency to solve problems or estimate probabilities based on information that is most readily available, or accessible, to their minds. Thus, Subject 405's assertion reflects her recent experience as a participant in the study rather than a logical approach toward problem solving (choice A is correct). The optimistic bias is people's tendency to assess the probability of personal success in future endeavors with more optimism than is warranted by actual facts (choice B is wrong). The representativeness heuristic refers to a problem-solving strategy by which people make choices based on their perception of which option is most characteristic of the issue in question. Since the question stem assumes that Subject 405 has no familiarity with kairomones other than through her participation in the study, there is no basis for Subject 405 to consider sulfur to be more representative of mammalian kairomones than any other substance (choice C is wrong). When people use logic or information-based arguments to persuade others, they are considered to be using the central route of persuasion. Subject 405, however, did not offer any such arguments to substantiate her claim (choice D is wrong).

The degradation of connective tissue components in emphysema is most likely caused by which of the following? A. Proteases B. Lipases C. Monoanime oxidases D. Glycoside hydrolases

A. The connective tissue components damaged in emphysema include collagen and elastin, which are extracellular proteins. Choice A is correct. Therefore, proteases released from the alveolar macrophages would be the culprits. Lipases degrade lipids (choice B can be eliminated), monoanime oxidases degrade certain neurotransmitters such as norepinephrine and epinephrine (choice C can be eliminated), and glycoside hydrolases degrade carbohydrates (choice D can be eliminated).

Which one of the following regions of the brain is most likely to be implicated in the onset and degenerative progression of Alzheimer's disease? A. Hippocampus B. Thalamus C. Posterior occipital lobe D. Superior parietal lobe

A. The hippocampus is associated with the formation, storage, and retrieval of memories. Since Alzheimer's often profoundly affects a sufferer's ability to remember, this region is most likely to be implicated in the onset and degenerative progression of AD (choice A is correct). The thalamus is a relay center for sensory information that is important for sensation and the perception of external stimuli. Since the primary symptom of Alzheimer's is memory loss, this is not the brain region most likely to be implicated (choice B is wrong). The posterior occipital lobe is critical to vision and the organization of visual information (choice C is wrong). The superior parietal lobe has many functions, including integration of spatial information and maintenance of internal representations; however, none of these functions involves the primary symptoms of AD (choice D is wrong).

A moon of mass m kg is orbiting a planet of mass M kg at a distance of R meters. If G denotes the universal gravitational constant, which one of the following expressions gives the moon's orbit speed? A. sqrt (GM/R) B. sqrt (2GM/R) C. sqrt (GMm/R) D. sqrt (GMm/R^2)

A. The moon is held in its circular orbit by the planet's gravitational pull, which provides the necessary centripetal force. Therefore, (mv^2/R)=(GMm/R^2) V^2= GM/R V= sqrt (GM/R)

Which of the following salts, when dissolved in water, would produce the solution with the highest pH? A. Na2CO3 B. NH4Cl C. NaCl D. AlBr3

A. The most basic salt will produce the solution with the highest pH. The carbonate ion from Na2CO3 is the conjugate base of bicarbonate, which is itself weakly basic in aqueous solution; choice A is the most basic salt listed. NaCl is a neutral salt as neither of its ions are reactive with water (eliminate choice C). Both NH4Cl and AlBr3 are acidic salts because both cations will interact with water to increase the [H+] in solution (eliminate choices B and D).

Pompe disease is most commonly characterized by excessive glycogen deposition in certain acidic cellular organelles containing the deficient enzymes In which cellular organelle are you most likely to find an abnormal accumulation of deposited glycogen in patients affected by Pompe disease? A. Lysosomes B. Mitochondria C. Golgi Body D. Nuclei

A. The passage states that the organelle lacking sufficient functional acid maltase and in which abnormal glycogen accumulates contains an acidic environment. Of the four choices, lysosomes have the most acidic environment making choice A true and the correct answer (eliminate choices B, C and D).

Alzheimer's disease, at all stages, is known to affect cognitive centers of the brain. Which of the following is NOT typically an early symptom of Alzheimer's? A. Sleep apnea B. Forgetfulness C. Losing or misplacing personal items D. Trouble with simple problem-solving tasks

A. The question stem states that Alzheimer's disease affects cognitive centers. Sleep apnea is a somatic condition in which breathing is temporarily disrupted during sleep; it is not a cognitive problem (choice A is not a symptom of Alzheimer's disease and is therefore correct). Pronounced forgetfulness is one of the first detectable symptoms of Alzheimer's disease and indicates impairment of the cognitive centers. For example, unlike the more commonplace forgetfulness that is often seen in older individuals, a man experiencing the early symptoms of Alzheimer's might forget the name of his grandchild or the state in which he lives (choice B is an Alzheimer's symptom and is therefore wrong). A tendency to frequently lose or misplace personal items is also an early symptom of Alzheimer's disease and suggests disrupted cognition (e.g., putting laundry into the kitchen cabinet; choice C is an Alzheimer's symptom and is therefore wrong). Alzheimer's disease also initially manifests itself in terms of an impaired ability to carry out simple tasks and to solve everyday problems. For example. a woman might find herself unable to follow the steps of a recipe that she has successfully followed many times before (choice D is also an Alzheimer's symptom and is therefore wrong).

Which of the following reflects the researchers' attempt to minimize adaptation effects during the experiment? A. Using wild mice rather than domesticated mice B. Using sugar water rather than pond water during the training stage C. Rearranging the bottles after a human subject detected sulfur D. Limiting the human subjects' inhalation time to two seconds

An adaptation effect is the phenomenon by which an organism becomes accustomed to a sensory stimulus, which results in a higher threshold for detection of that stimulus. Since the experiment in this study was designed to measure detection thresholds, it was necessary to limit participants' inhalation time to two seconds so that they would not become accustomed to the smell of sulfur. Otherwise, their olfactory thresholds for the detection of sulfur may have measured higher than their actual levels (choice D is correct; choices A, B, and C are wrong).

Researchers used a probability-based sample to recruit non-physician participants, which lead to a non-physician sample representative of the national population. Therefore, even though demographic information is not presented in the passage, it is still possible to conclude that: I. the most common religious affiliation reported is Christian (as compared with Muslim, Jewish, Hindu, and Buddhist). II. the least common socioeconomic status reported is lower class (as compared with upper class, middle class, and working class). III. for married participants, the most common relationship structure reported is monogamous (as compared with polygamous). A. I and II only B. I and III only C. II and III only D. I, II, and III

B. Item I is true: the most common religion in the United States is Christianity (choice C can be eliminated). Christianity is the largest religion in the world, and in the United States 80% of the people identify as Christian. Item II is false: the least common socioeconomic status in the United States is upper class, not lower class (choices A and D can be eliminated). In the United States, the top 3% of the population is considered upper class, while roughly a quarter of the population is considered lower class. Item III is true: the most common form of marriage in the United States is monogamy. The social norm in the U.S. is monogamous marriages, in which the partners are married only to each other. Furthermore, this convention is reinforced through legislation, as polygamous marriages are illegal in the United States (choice B is correct).

According to the laws of thermodynamics, an endothermic reaction is most likely to be spontaneous if the: A. change in entropy is positive and the temperature is low. B. change in entropy is positive and the temperature is high. C. change in entropy is negative and the temperature is high. D. change in entropy is negligible and the temperature is low.

B. An endothermic reaction has a positive ΔH (eliminate choices C and D). Since a reaction is spontaneous if ΔG = ΔH - TΔS is negative, an endothermic reaction is spontaneous when ΔS is positive and the temperature is high (eliminate choice A and choice B is correct).

Novobiocin is an antibiotic that inhibits bacterial DNA gyrase. Its antibiotic properties are based on the fact that it: A. prevents DNA helix formation. B. inhibits DNA replication. C. prevents RNA transcription. D. inhibits protein translation.

B. DNA gyrase supercoils bacterial DNA (choice B is correct). It does not affect helix formation; the DNA is already in a double-helix before it is supercoiled (choice A is wrong). It does not affect protein synthesis, since that has to do with RNA and ribosomes, not DNA (choice D is wrong). The job of DNA gyrase is to continually introduce negative supercoils into the circular bacterial genome. Then, when the helix is opened for replication and positive supercoils are introduced (the DNA gets more tightly wound at the ends of the replication fork), the already-present negative supercoils cancel out the newly introduced positive supercoils, and DNA tension is kept to a minimum. If this does not occur, the DNA strand would become too tightly wound for replication to continue, and in the absence of replication, the bacteria cannot reproduce. Note that the same could be argued for transcription, but since that involves such a small region of the genome, the positive supercoiling is not a significant effect (choice C is wrong).

The precursor for serine biosynthesis, glyceraldehyde-3-phosphate, is produced from fructose-1,6-bisphosphate. Fructose-1,6-bisphosphate is produced in which of the following processes? A. β-oxidation of fatty acids B. Glycolysis C. Amino acid loading D. Krebs Cycle

B. Fructose-1,6-bisphosphate is produced in glycolysis in a reaction catalyzed by phosphofructokinase (choice B is correct). It is not produced in any of the other processes (β-oxidation of fatty acids, amino acid loading, or the Krebs cycle) so eliminate choices A, C and D.

A child who learns to avoid approaching her parent for help and comfort because she is always met with aggression by the parent develops an anxious-avoidant insecure attachment style. The child thus associates the parent with fear and begins to actively avoid interactions with the parent in order to avoid this emotional experience. This type of learning can also be explained by which operant conditioning principle? A. Modeling B. Avoidance learning C. Vicarious learning D. Role-playing

B. In avoidance learning, an organism's behavior is reinforced by the termination or prevention of an aversive stimulus. In other words, the individual is motivated to escape fear-producing environmental stimuli or conditioned stimuli. In this instance, because the child is always met with anger when she encounters the parent, the parent becomes a conditioned stimulus that produces fear. The child thus begins to actively avoid interactions with the parent in an effort to avoid experiencing fear (choice B is correct). Observational learning, also known as modeling or vicarious learning, is part of Albert Bandura's social learning theory. Bandura believed that learning could occur by watching others. However, not all observed behaviors are effectively learned. In order for the modeling process to work, according to Bandura, the following factors have to be present: attention, retention, reproduction, and motivation. The child in question is avoiding the parent and does not appear to be motivated to reproduce the aggressive behavior that she is observing (choices A and C are wrong). Role-playing is not an operant conditioning principle. Rather, it is an instance or situation in which one deliberately acts out or assumes a particular character or role. In psychology, it is a therapeutic technique designed to reduce conflict in social situations; participants act out particular behavioral roles in order to expand their awareness of differing points of view. Role-playing has also been the focus of many social psychology studies (choice D is wrong).

Research has also considered the salaries of medical professionals and the effects of compensation on overall life satisfaction. Reports of average earnings range from $156,000 to $315,000, with pediatricians reporting the lowest salaries and orthopedic surgeons and radiologists reporting the highest salaries. Based on this information, models of social stratification in the United States would consider the average anesthesiologist to be a member of which social class? A. Upper class B. Middle class C. Working class D. Lower class

B. Most sociologists consider socioeconomic status (SES) when discussing social stratification. This measure is often defined in terms of power, prestige, and wealth owing to the common interrelationship among these factors. Despite the high earnings of these medical specialists in comparison to the national average, it is a common misconception that these professionals are "upper class". According to most models of social stratification, the income range for upper class is much higher. Upper class annual incomes range from millions to billions of dollars

The acidity of hydrogen halides increases as one moves down the periodic table. Which of the following properties of halides is most useful in explaining this trend? A. Boiling point B. Ionic radius C. Density D. Atomic weight

B. Moving down the periodic table, the ionic radii of the atoms increase. As the ionic radius increases, the stability of the corresponding anion also increases (since the negative charge is distributed over a larger volume). Therefore, since larger halides are more stable as anions, larger hydrogen halides are more likely to undergo deprotonation, and are stronger acids.

Which of the following describes how the brain uses parallel processing to process a visual stimulus? A. A different area of the brain is activated by each type of image (images, letters, etc.). B. Visual areas V1-V5 are utilized to analyze different aspects of an image simultaneously. C. Images are simultaneously processed using both top-down and bottom-up processing. D. Visual areas V1-V5 are utilized to analyze each aspect of an image in succession.

B. Parallel processing describes the simultaneously processing of different aspects of a stimulus (choice B is correct), instead of analyzing the pieces of a stimulus step-by-step (choice D can be eliminated). Choice A is the definition of feature detection theory, and can be eliminated. Top-down and bottom-up processing are elements of gestalt psychology; the simultaneous use of both top-down and bottom-up processing is characteristic of most sensory processing, but is not the definition of parallel processing (choice C can be eliminated).

Which of the following would be true about cis-oleic acid, a monounsaturated fatty acid with the formula CH3(CH2)7(CH)2(CH2)7COOH? A. It will generate approximately 119 ATP after 9 rounds of β-oxidation, followed by the Krebs cycle and the electron transport chain. B. It will generate approximately 119 ATP after 8 rounds of β-oxidation, followed by the Krebs cycle and the electron transport chain. C. It will generate 90 ATP after 9 rounds of β-oxidation, followed by the Krebs cycle and the electron transport chain. D. It will generate 90 ATP after 8 rounds of β-oxidation, followed by the Krebs cycle and the electron transport chain.

B. Recognize the ability to treat this question as a 2x2 elimination. cis-Oleic acid has 18 carbons and thus will undergo 8 rounds of β-oxidation (eliminate choices A and C). This will generate 9 molecules of acetyl-CoA, 8 molecules of NADH and 7 molecules of FADH2 since it is a monounsaturated fatty acid. Each of the 9 acetyl-CoAs will go through the Krebs cycle and this will generate 27 NADH (which will give 67.5 ATP), 9 FADH2 (which will give 13.5 ATP) and 9 GTPs (9 ATP equivalents). This means the acetyl-CoAs alone generate 90 ATP equivalents. Since the NADH and FADH2 made in β-oxidation will generate even more ATP, the total will exceed 90 (eliminate choice D and choice B is correct). The 8 molecules of NADH made in β-oxidation will lead to 20 ATP and the 7 FADH2 will give 10.5. Also remember that fatty acid activation (which must occur before β-oxidation) costs the cell two high energy bonds, or ATP equivalents. This means the electron carriers made in β-oxidation will give a net yield of 28.5 ATP. Overall then, cis-oleic acid will generate 118.5 ATP molecules.

The low chemical reactivity of nitrogen gas is most likely caused by the: A. low oxidation state of each nitrogen atom in the molecule. B. triple bond holding each nitrogen atom in the molecule. C. low electronegativity of each nitrogen atom in the molecule. D. small size of each nitrogen gas molecule.

B. Since the two nitrogen atoms participate in a strong triple bond, a great deal of energy is required to dissociate them, resulting in their low reactivity.

A scientist is experimenting with enzymes found in the human body. Of the following, which would make the most appropriate buffer for such work? A. C6H5COOH (Ka = 6.5 × 10-5) B. HClO (Ka = 3.5 × 10-8) C. HCN (Ka = 4.9 × 10-10) D. H3CNH3+ (Ka = 2.2 ×10-11)

B. The human body has a pH of 7.4. It is best to choose a buffer whose pKa value is close to the pH that one is trying to maintain. Since HClO has a Ka value of 3.5 × 10-8, we know that its pKa is a little less than 8 (in fact, it's about 7.5). Therefore, of the choices listed, HClO (choice B) would be the best here.

Antibodies specific to the mitotic spindle apparatus would most likely recognize products of: A. transcription. B. translation. C. transformation. D. replication

B. The spindle is composed of microtubules, which are polymers of tubulin protein monomers. Antibodies against the spindle would recognize proteins, which are produced during translation (choice B is correct). Choices A, C, and D are wrong because these processes do not produce proteins.

There are five recognized tastes: sweet, salty, bitter, sour, and umami. Recent research suggests there is also a sixth, oleogustus, the unique taste of fat. The lead researcher on the study said that since there aren't any words that exist for this taste, they were forced to make it up, and it was very difficult to figure out if people really view this as unique sensation. This best demonstrates: A. gestalt principles. B. the Sapir-Whorf hypothesis. C. major limitations of sensory processing. D. monocular cues.

B. This example suggests that it was difficult to describe and define things for which we don't have language; this is an example of the Sapir-Whorf hypothesis, which suggests that the structure of a language determines/influences the thought and behavior characteristic of the culture in which it is spoken. In other words, without an actual word to define the fatty taste, it was difficult to determine that it was in fact a unique taste (choice B is correct). Gestalt psychology emphasizes the mental processes that perceive the "whole" of objects or situations. This school of psychology has been particularly influential in describing tendencies of human visual perception. Gestalt principles do not describe how a lack of language con influence perception (choice A is wrong). This example describes a major limitation of language to define a sensation, not a limitation of sensory processing itself (choice C is wrong). Monocular cues, such as relative size and texture gradient (among others) are cues that can be perceived with only one eye that help us judge distance (depth perception). They have nothing to do with taste (choice D is wrong).

Research indicates that professional burnout can result in thoughts of suicide. Suicidal ideation is most often associated with a psychological disorder in which broad category? A. Anxiety disorders B. Mood disorders C. Personality disorders D. Psychotic disorders

B. Thoughts of death or suicide are most often associated with major depressive disorder, which is a mood disorder characterized by persistent feelings of sadness and hopelessness (choice B is correct). Mood disorders involve disturbances in mood or affect. Anxiety disorders involve excessive worrying and include generalized anxiety disorder, panic disorder, and obsessive-compulsive disorder (choice A is wrong). Research does not show a significant association between these disorders and suicidal ideation, with the exception of post-traumatic stress disorder. Personality disorders involve enduring maladaptive patterns of behavior and cognition and include antisocial personality disorder, avoidant personality disorder, and borderline personality disorder (choice C is wrong). Psychotic disorders involve a loss of contact with reality and include schizophrenia and delusional disorder (choice D is wrong).

Hate crimes committed against immigrants are a consequential example of: A. homophobia. B. xenophobia. C. a social phobia. D. a specific phobia.

B. Xenophobia is defined as the fear of that which is perceived to be foreign, such as the fear of cultural outsiders (choice B is correct). Xenophobic attitudes are one of the negative reactions to immigration, whether involving genuine physical fear or concerns about economic, political, or social factors (such as increased competition for jobs). Homophobia is more complex and is defined as the wide range of negative thoughts and attitudes about non-heterosexual individuals, as well as any resultant behavior (choice A is wrong). A social phobia is defined as the fear of potential embarrassment or humiliation in public situations (choice C is wrong). For example, the fear of using a public restroom is a common example of a social phobia that can influence one's ability to perform routine activities. A specific phobia is defined as the fear of a specific object or situation (choice D is wrong). These are divided into four categories based on the specific triggers: animal, medical, natural environmental, and situational phobias (e.g., arachnophobia is a common specific animal phobia of spiders).

What substance might be expected to be in mosquito saliva that could inhibit the process of hemostasis? A. A vasoconstrictor B. An inhibitor of platelet aggregation C. A stimulator of coagulation D. A stimulator of T cells

B. hemostasis = stoppage of blood flow. so inhibiting hemostasis means allowing blood to flow/not stopping blood flow. Platelets are an essential part of the hemostatic process; they form large clumps (aggregation) as part of clot formation. Thus an inhibitor of platelet aggregation could inhibit hemostasis (choice B is correct). Vasoconstrictors constrict blood vessels to slow bleeding and can contribute to hemostasis (choice A is wrong). Coagulation is the process by which clots form to stop bleeding; a stimulator of coagulation would not inhibit hemostasis (choice C is wrong). T cells do not play a role in coagulation (choice D is wrong).

Though the bond between parent and child begins at birth, attachment patterns can be better identified once the child becomes mobile and is able to explore his or her surroundings. Thus, birth to 18 months is a critical time in which attachment bonds form and in which attachment styles begin to be observed and solidified. Erik Erikson's first stage of development, which also encompasses birth to 18 months, coincides nicely with attachment theory and its tenets. What is the basic conflict in Erikson's first stage? A. Initiative vs. Guilt B. Industry vs. Inferiority C. Trust vs. Mistrust Correct Answer D. Autonomy vs. Shame and Doubt

C

Suppose that Sarah is kneading a piece of clay in her hands. Which of the following is NOT a strategy relevant to conservation? A. Sarah realizes that the clay is the same piece of clay, regardless of its shape. B. Sarah notices that she can shape the clay into a ball, then a pancake, then back into a ball again. C. Sarah knows that the clay still exists even when her mother puts it back into its container. D. Sarah discovers that squeezing the clay in the middle forces it to become bulkier on the sides.

C. The realization that an object continues to exist despite its disappearance from the visual field is known as object permanence (choice C is not a strategy relevant to conservation and is therefore correct). Conservation is the term that Piaget used to describe children's recognition of constancy and consistency in physical matter despite changes in shape or container. This awareness is characteristic of the stage of concrete operational thought, which is usually achieved between the ages of 6 or 7 and 11 or 12. Conservation is realized through three insights: identity, reversibility, and reciprocity. Identity involves the child's recognition that the physical object is the same object, regardless of how it is manipulated (choice A is a strategy relevant to conservation and is therefore wrong). Reversibility involves the child's recognition that a given manipulation can be reversed to give the object its initial appearance (choice B is a strategy relevant to conservation and is therefore wrong). Finally, reciprocity involves the child's realization that a manipulation of one dimension or aspect yields a corresponding change in another aspect. As a result, the manipulation is understood to change the original object rather than create a new one (choice D is a strategy relevant to conservation and is therefore wrong).

N2(g) + 3 H2(g) --> 2 NH3(g) The continual removal of the NH3 from the reaction chamber maximizes the reaction yield because: A. the large volume occupied by ammonia gas decreases the available volume for the remaining reactants, thereby increasing the pressure in the reaction chamber. B. the ammonia gas will eventually react with the free hydrogen, creating ammonium ions and reducing the amount of reactants available to the reaction. C. removing a product as it forms creates a stress on the reactants side of the equilibrium, causing the reaction to produce more ammonia. D. the presence of excess ammonia increases the temperature in the reaction chamber, leading to a decrease in the rate of product formation.

C. A reaction can be driven forward by the addition of starting materials or the removal of products to perturb the equilibrium. This is an example of Le Châtelier's principle.

During peak cardiac activity, heart muscle utilizes the anaerobic pathway for only 10% of its energy as opposed to up to 85% for maximally contracting skeletal muscle. With respect to skeletal muscle, cardiac muscle would: A. convert more pyruvate to lactate. B. consume 1/8 as much ATP to produce the same contractile force. C. consume less glucose per ATP produced. D. require less oxygen per molecule of ATP produced.

C. Aerobic respiration includes glycolysis, the PDC, the Krebs cycle, and electron transport, to produce a total of 32 ATP per glucose (assuming the malate-aspartate shuttle is used to transfer the electrons from glycolytic NADH into the mitochondria). By contrast, anaerobic respiration involves only glycolysis and fermentation of pyruvate to lactate or alcohol, to produce 2 ATP per glucose. Anaerobic respiration is therefore much less efficient and must consume much more glucose to produce the same amount of ATP. Since cardiac muscle uses mostly aerobic respiration, it will be more efficient than skeletal muscle and uses less glucose to produce an equivalent amount of ATP (choice C is correct). Conversion of pyruvate to lactate occurs during fermentation, which cardiac muscle carries out less of, not more (choice A is wrong). The amount of contractile force generated per ATP is independent of the source of ATP used (choice B is wrong). Cardiac muscle uses more aerobic respiration, so it will use more oxygen per ATP produced, not less (choice D is wrong).

99mTc has a half-life of about 6 hours, emitting 140 keV gamma rays as it decays to 99Tc, a beta emitter with a half-life of 211,000 years. Which of the following is NOT a good justification for the use of 99mTc in nuclear medicine? A. Its short half-life limits its negative biological impacts. B. The long half-life of its decay product (99Tc) compared to that isotope's one day "biological half-life" (the time it takes for half of a substance to lose half of its biological potency due to metabolic effects) means that the decay product poses little additional radiation risk. C. Gamma radiation is not ionizing and so poses no medical risk. D. The predictable frequency of the emitted photons simplifies detection and imaging.

C. All forms of nuclear radiation—alpha, beta, and gamma—are ionizing to some extent, so choice C is a false statement and therefore the correct answer. All of the other choices are plausible (and indeed true).

It takes the oldest Maaza brother 8.0 seconds to swing out on a trapeze from the platform and return. In a certain stunt one of the brothers hangs from the ankles of the oldest brother as the oldest brother swings from the platform and returns. This stunt requires approximately how long, given the equation for the period of oscillation of a simple pendulum is given by T = 2π√(L/g) ? A. 4.0 seconds. B. 7.8 seconds. C. 8.3 seconds. D. 16.0 seconds.

C. Approximating the center of mass of any of the brothers to be located at their midpoint, the effective length of the pendulum increases from 15 + 1 = 16 m with just one Maaza brother to 15 + 1 + 1 = 17 m with two. T= 2pi sqrt (L/g) this slight increase in length will create a slight increase in the period of the trapeze (pendulum). Since the period is 8.0 seconds with just one Maaza, choice C (8.3 seconds) is the best response here.

Given (Ka = 1.8 x 10-5) What is the pH of a solution containing equal concentrations of acetic acid and acetate ion? A. 1.8 B. 3.5 C. 4.7 D. 5.2

C. As the passage states, when the concentration of the acid (acetic acid) is equal to the concentration its conjugate base (acetate ion), the pH of the solution will be equal to the pKa of the acid. So pH = pKa = -log Ka = -log (1.8 × 10-5) = 5 - log 1.8 ≈ 4.7.

All of the following processes occur during the conduction of a nerve impulse across the synapse of two neurons EXCEPT: A. the release of vesicles from the presynaptic cell. B. the opening of postsynaptic ion channels. C. a retrograde depolarization of the presynaptic cell. D. the depolarization of the postsynaptic cell.

C. During the conduction of an action potential across a chemical synapse, neurotransmitter is released by the presynaptic cell, it diffuses across the synapse, binds to receptors on the postsynaptic cell, opens ion channels on the postsynaptic cell, and depolarizes (or hyperpolarizes) the postsynaptic cell. Thus, choices A, B, and D are all part of the normal conduction of an action potential across a synapse and can be eliminated. The presynaptic cell does not normally transmit a retrograde action potential, however (choice C is false and the correct response here). This is usually prevented by the refractory period after an action potential passes, causing action potentials to propagate in only one direction along an axon.

Which of the following conditions leads to a reduction in water reabsorption by the kidney? A. High plasma levels of vasopressin B. Increased osmolarity of interstitial fluid in the kidney medulla C. Decreased permeability of the collecting duct to water D. An ACTH-secreting tumor

C. Factors that reduce water resorption would tend to increase urinary output, decrease blood volume, and decrease blood pressure. High levels of vasopressin (ADH) would increase water resorption and plasma volume (choice A is wrong). The greater the osmolarity of the interstitial fluid in the medulla of the kidney, the greater the water resorption as filtrate passes into the medulla in the descending loop of Henle (choice B is wrong). As filtrate passes through the collecting duct, water can be reabsorbed and the urine concentrated. If the collecting duct is impermeable to water, however, then less water will be reabsorbed, and the filtrate will remain dilute (choice C is correct). ACTH is the hormone that controls secretion of aldosterone (as well as cortisol) by the adrenal cortex. An ACTH-secreting tumor will cause elevated secretion of aldosterone, increased Na+ reabsorption, and a subsequent increase in water reabsorption (choice D is wrong).

There are multiple sociological perspectives on deviance. Which of the theorists below are expected to consider the associated questions about non-normative behaviors? I. Differential association theorists; "How can people resist deviance?" II. Labeling theorists; "Who defines deviance?" III. Structural strain theorists; "How do norms affect deviance?" A. III only B. I and II only C. II and III only D. I, II, and III

C. Item I is false: differential association theorists argue that deviant behaviors are learned through interactions between individuals and their communities. These theorists contend that people become deviant when they are exposed to deviant behavior and attitudes. "How can people resist deviance?" is therefore not a pertinent question (choices B and D can be eliminated). The main criticism of differential association is that it essentially reduces individuals to their environments. Because it states that those in deviant communities learn to be deviant themselves, the possibility of resistance is not contemplated. Item II is true: labeling theory asserts that behaviors are seen as deviant as the result of social processes of labeling. "Who defines deviance" is therefore a pertinent question (choice A can be eliminated). Labeling theorists do indeed address this concern; for example, these theorists are interested in the mechanisms through which power contributes to deviance labeling (e.g., agents of social control). Item III is true: structural strain theory suggests that deviant behaviors are the result of tension between the accepted social goals and the institutionalized means available to achieve those goals. "How do norms affect deviance?" is therefore a pertinent question (choice C is correct). Structural strain theorists consider the effects that social norms have on behavior, normative or not, and suggest that there is pressure to use deviant methods when the social structure does not support the achievement of the accepted goals.

Which of the following are determinants of the G2/M checkpoint? Ensure that genomic replication is complete Take inventory of nucleotide levels Check for mutations or DNA instability A. II only B. I and II only C. I and III only D. I, II, and III

C. Item I is true: Before the cell divides, it must check the integrity of the genome it is passing to the daughter cells. This includes making sure DNA replication is complete. Choice A can be eliminated. Item II is false: Checking the levels of nucleotides in the cell is part of the G1/S checkpoint (the other major checkpoint pathway mentioned in the passage), since these building blocks are required for DNA replication (choices B and D can be eliminated). Item III is true: Before the cell divides, mutations have to be repaired. Choice C is correct.

The function of galactoside permease is to transport lactose across the bacterial cell membrane. Galactoside permease is necessary because: A. lactose is hydrophobic and cannot cross the lipid bilayer. B. lactose needs a protein carrier to move it against its concentration gradient. C. lactose is hydrophilic and cannot cross the lipid bilayer. D. lactose needs to be converted to allolactose to enter the cell.

C. Lactose, like other carbohydrates, is very hydrophilic, not hydrophobic (choice A is wrong). Hydrophilic molecules are unable to diffuse across a membrane into the cell (choice C is correct). There is no information to support B; in any case, it can be assumed that lactose is not being transported against a gradient (choice B is wrong). Allolactose only signals the presence of lactose, but it is lactose that is the energy source, and it is lactose that is imported into the cell (choice D is wrong).

One advantage of using a longitudinal study design, such as the one described in the passage, is: A. the ability to conduct in-depth analyses of a few participants to get very rich and detailed data samples. B. the opportunity to follow a cross-section of participants of different types at the same time. C. the opportunity to follow one group of participants over an extended period and track their changes. D. the ability to see how many participants remain in the study and how many people withdraw.

C. Longitudinal studies in developmental psychology research, such as the one in the passage, allow researchers to follow one group of participants over time. In this study, they were able to follow participants over a six-year period and track their changes (choice C is correct). Case study design (not a longitudinal study design) allows researchers the opportunity to conduct in-depth analyses of a few select participants, then emerge with very detailed data (choice A is wrong). Cross-sectional research (not a longitudinal study design) gives researchers a variety of participants of different types (e.g., different ages) that they can track at the same time (choice B is wrong). A drawback (not an advantage) of longitudinal research is that sometimes participants will withdraw from the research over the period of the study for a variety of reasons, including moving away, lack of interest, or even illness or death (choice D is wrong).

A secure attachment style is best demonstrated by which of the following scenarios? A. A five-year-old child who remains by her mother's side at a children's birthday party, not interacting with the other children B. A four-year-old child who runs toward her mother with open arms upon her return from work, but before reaching her turns around and instead runs away frightened. C. A four-year-old who confidently joins in a wide array of activities while at a birthday party and later seeks his mother's help, and is soothed by her, when a game of tag results in a skinned knee D. A five-year-old child who explores different parts of a playground, pushing his climbing of the jungle gym to dangerous heights, with the knowledge that his mother will not be paying attention

C. Secure attachment is promoted by a responsive and available parent and is characterized by a child who is able to explore his or her environment secure in the knowledge that the parent will be there when needed. A child confidently exploring and joining in different activities at a birthday party demonstrates a secure attachment style. The child's act of seeking his mother's aid during a stressful time, such as when being injured, and the parent being available to comfort and soothe him, further displays secure attachment (choice C is correct). Reluctance to explore and instead clinging and remaining by the mother's side is indicative of an anxious-resistant insecure attachment style, in which the child is uncertain about the parent's availability because of previous separations. Because of this fear of being abandoned, he or she stays close and clings to the mother (choice A is wrong). Odd or awkward behaviors when separating or reuniting with the parent, such as extreme shifts between proximity-seeking and avoidance, are indicative of a disorganized attachment style. A child who, upon being reunited with her mother, initially runs toward her with open arms and then quickly runs away instead illustrates an extreme shift between proximity-seeking and avoidance. Disorganized attachment is usually the result of the child experiencing trauma, such as being physically, emotionally, or sexually abused by the parent. In this instance, the child sees the parent as both frightening and the only source of comfort, which causes great confusion and results in the eratic proximity-seeking and avoidance behavior (choice B is wrong). An anxious-avoidant attachment style results from the child being constantly rejected and rebuffed by the parent. These children learn to be independent early on, depending only on themselves because they learn that they cannot rely on the parent who is never available. A child exploring and climbing the jungle gym to dangerous heights displays this early independence. The child's knowledge about the lack of availability and responsiveness of the parent further provides evidence of an anxious-avoidant attachment style (choice D is wrong).

A bus is traveling at 25 m/s, and a cyclist is traveling at 5 m/s behind the bus, in the same direction. The frequency of a sound coming from the bus is observed by the cyclist to be 1000 Hz. Approximately what is the frequency of the sound as perceived by the bus driver? A. 950 Hz B. 1000 Hz C. 1060 Hz D. 2100 Hz

C. Since the bus is traveling away from the cyclist faster than the cyclist is traveling toward it, there is a steady increase in the distance between the source and detector. Motion "away" always corresponds to a frequency decrease. Thus, if the cyclist (detector) hears a frequency of 1000 Hz, the actual source frequency must be higher than 1000 Hz; this eliminates choices A and B. No calculation is really necessary to decide between choices C and D, because for detector and source speeds much less than the speed of sound, the change in frequency due to the Doppler Effect will always be a percentage much less than 100%. Thus choice D is too high a frequency to be correct here.

Larry is a hardworking student who always earns high grades. When he is assigned to work on a group project with four of his classmates, however, his motivation seems to dwindle. Instead of pressing on for hours as he normally would for an individual assignment, he develops a lackadaisical attitude toward the project. Which of the following can most accurately explain his change in attitude? A. Social facilitation B. Assimilation C. Social loafing D. Peer pressure

C. Social loafing occurs when individuals exert less effort than they normally would because they are working in a group setting. In this scenario, submaximal goal-setting is apparent in that Larry does not expect that he will need to do as much work since others will be working toward the final outcome as well (choice C is correct). Like social loafing, social facilitation takes place in group settings. However, social facilitation occurs when individuals perform simple or well-practiced tasks better when they know others are watching (choice A is wrong). In sociology, assimilation is the process by which individuals, often immigrants, become absorbed into their new society (choice B is wrong). Peer pressure occurs when an individual feels pressured by his or her peers to act a certain way; there is no indication in the question stem that Larry's group members suggested that he put in less effort (choice D is wrong).

What is the main premise of the behaviorist theory of personality? A. Personality is the result of a person's multiple traits. B. Personality is the result of a person's unconscious mind and childhood experiences. C. Personality is the result of interactions between the person and the environment. D. Personality is the result of a person's genetics.

C. The behaviorist theory of personality emphasizes that personality is derived from the interactions between a person and his or her environment (choice C is correct). Trait theory, a type of humanist theory, emphasizes that personality is the result of a person's multiple traits that are relatively stable over time (choice A is wrong). Psychodynamic theories emphasize that personality is the result of a person's unconscious mind and childhood experiences (choice B is wrong). Biological theories of personality emphasize that personality is the result of a person's genetics (choice D is wrong).

The function of the electron transport chain is to transfer the energy stored in electrons. The primary source of high-energy electrons is: A. ATP. B. the mitochondrial pH gradient. C. NADH. D. NADPH.

C. The high-energy electrons that enter electron transport originate from the NADH produced in the Krebs cycle (choice C is correct). The electron transport chain creates the mitochondrial pH gradient (choice B is wrong), which is used to make ATP in turn (choice A is wrong). NADPH is structurally almost identical to NADH but has very different functions. NADPH is used as a source of biosynthetic energy in processes like fatty acid biosynthesis, but it is not used as a source of high-energy electrons by the electron transport chain (choice D is wrong).

Hydrolysis of the phosphocreatine bond releases 43.1 kJ/mol of energy. What is the standard free energy of hydrolysis for ATP? A. -87.4 kJ/mol B. -43.1 kJ/mol C. -30.5 kJ/mol D. +6.7 kJ/mol

C. The passage says that creatine kinase can transfer a phosphate group from phosphocreatine to ADP via a spontaneous reaction. In other words, phosphocreatine dephosphorylation can be coupled to ADP phosphorylation and the net overall reaction is spontaneous, or has a -ΔG. If phosphocreatine dephosphorylation releases 43.1 kJ/mol, ADP phosphorylation must use less energy than this (choice C is correct). If ATP hydrolysis released 87.4 kJ/mol or 43.1 kJ/mol, the transfer of a phosphate group from phosphocreatine to ADP would not be a spontaneous reaction (choices A and B are wrong). ATP hydrolysis releases energy to the cell, so must have a negative standard free energy (choice D is wrong).

What is the FRC for an individual with the following lung volumes: total lung capacity (TLC) = 5700 mL, vital capacity (VC) = 4700 mL, inspiratory reserve volume (IRV) = 3000mL and tidal volume (VT) = 500 mL? A. 3500 mL B. 1000 mL C. 2200 mL D. 1200 mL

C. The passage states that FRC = ERV + RV. TLC = VC + RV SO....RV= TLC - VC = 5700 mL - 4700 mL = 1000 mL VC = VT + IRV + ERV SO.... ERV = VC - VT - IRV = 4700 mL - 500 mL - 3000 mL = 1200 mL FRC = ERV + RV = 1200 mL + 1000 ml = 2200 mL

Which of the following best describes the cascade that leads to cellular apoptosis after exposure to intra- or extracellular death signals? A. Effector kinases cluster together and activate each other. The activation of effector kinases leads to the activation of initiator caspases, which cleave proteins at aspartic acid sites, triggering apoptosis. B. Effector caspases cluster together and activate each other. The activation of effector caspases leads to the activation of initiator caspases, which cleave proteins at aspartic acid sites, triggering apoptosis. C. Initiator caspases cluster together and activate each other. The activation of initiator caspases leads to the activation of effector caspases, which cleave proteins at aspartic acid sites, triggering apoptosis. D. Effector kinases cluster together with initiator capsases, activating the initiator capsases. The activation of effector kinases leads to the deactivation of initiator caspases, which cleave proteins at aspartic acid sites, triggering apoptosis.

C. This is a recall question pertinent to concepts discussed in the passage (apoptosis). Apoptosis is carried out by proteases called caspases, which cleave targets at aspartic acid residues; apoptosis is not started by effector kinases (eliminate choices A and D). Initiator caspases cluster together and initiate apoptosis (choice C is correct). These initiator caspases can then activate effector caspases, which cleave proteins at aspartic acid residues; this triggers apoptosis. Choice B is incorrect, and confuses the role of initiator and effector caspases.

During the blastula stage, neighboring cells are in close contact with each other. The formation of the blastula is contemporaneous with the formation of tight junctions. The function of tight junctions is to: A. provide a means through which induction occurs. B. act as pores connecting the cytoplasm of adjacent cells. C. seal off the blastocoel and unite connected cells into a tissue. D. transmit action potentials between cells.

C. Tight junctions hold cells together and can form a seal so that fluid does not leak between cells (choice C is correct). Induction involves communication between cells but can be performed by secreted factors and does not require tight junctions for the communication to happen (choice A is wrong). Tight junctions do not, however, form a junction between the cytoplasm of cells (choice B is wrong), nor do they allow the transmission of action potentials (choice D is wrong).

While walking the tightrope, the youngest Maaza brother carries a long heavy pole as shown in Figure 2. What is the most likely reason for this? A. The extra mass of the pole stabilizes the cable by creating more tension. B. The extra mass of the pole lowers his center of gravity making balance easier. C. The long pole increases his rotational inertia making balance easier. D. The long pole decreases his rotational inertia making balance easier.

C. While the extra mass does create more tension in the cable, it is not clear how this "stabilizes" it or makes balancing on it any easier, so A is not the best choice. Since Figure 2 shows the young Maaza holding the pole at roughly the level of his own center of mass, the additional mass of the pole would not change the overall center of mass by very much, so B is not the best choice. As for C and D, the pole can only increase his rotational inertia, so D is eliminated. Choice C is indeed the best choice here since the increase in rotational inertia will imply less tendency to rotate (that is, less tendency to topple off the cable), thus making balancing easier.

Which of the following statements demonstrates the idea that mass media are agents of socialization? Young people's body image is affected by the bodies depicted in magazines News media coverage of sexual assault focuses primarily on the victim Social media has become a pervasive aspect of everyday life A. I only B. I and II only C. II and III only D. I, II, and III

D

The conformational change of a regulatory protein after the binding of a repressor most likely represents an alteration of the protein's: A. amino acid composition. B. primary structure. C. secondary structure. D. tertiary structure

D. A conformational change in a protein is a change in the larger scale folding of a protein, with changes in the relative positions in space of amino acids located far from each other in the linear polypeptide chain. The tertiary structure of a protein involves large-scale structure within a polypeptide chain that is stabilized by interactions between amino acids that can be distant from each other in the linear sequence; thus, this is the most likely level of structure altered during a conformational change (choice D is correct). The amino acid composition and primary structure (the linear sequence of amino acid residues in the polypeptide chain) cannot be changed without breaking covalent peptide bonds, which does not occur in a conformational change (choices A and B are wrong). The secondary structure of a protein involves folding that is stabilized by the nearest neighbors in the polypeptide chain, including structures such as α helix and β sheet. Conformational changes can alter secondary structures somewhat, but mostly alter tertiary structure (choice D is a better answer than choice C).

After amino acids are synthesized, they can be joined to form proteins. Which of the following best describes the type of bond used to join amino acids? A. Carbonyl bond B. Phosphodiester bond C. Hydrogen bond D. Peptide bond

D. A peptide bond describes the type of bond formed between two adjacent amino acids (choice D is correct). There is no such bond as a carbonyl bond (choice A is eliminated). A phosphodiester bond is a covalent bond formed between a phosphate group and a pentose sugar. This is the type of bond used to attach the phosphate backbone to nucleotides (choice B is incorrect). A hydrogen bond is an electromagnetic interaction between two polar molecules; it is not a covalent bond, like a peptide bond (choice C is incorrect).

Within animal cells, the transport of Na+ out of the cell by the Na+/K+ ATPase involves: A. a symport. B. an antiport. C. facilitated diffusion. D. active transport

D. An ATPase uses the energy of ATP hydrolysis to drive the transport of Na+ out of the cell against its electrochemical gradient, thus it is an example of active transport (choice D is correct). Symports, antiports, and facilitated diffusion (choices A, B, and C) all involve the movement of ions down a gradient, which is not active transport.

In a male individual with Down's Syndrome (trisomy 21), how many chromosomes would be visible at metaphase I of spermatogenesis? A. 23 B. 24 C. 46 D. 47

D. An individual with trisomy 21 has an extra copy of chromosome 21 (three total copies). During metaphase I, the developing gametes are still diploid (separation of homologues has not yet occurred), so this individual would have the normal 46 chromosomes plus the extra copy of chromosome 21, for a total of 47 chromosomes (choice D is correct; eliminate choices A, B and C).

The enzyme lysozyme degrades peptidoglycan. If lysozyme is mixed in blood agar prior to addition of Strep. viridans, which of the following would be observed? A. Colonies surrounded by a clear zone B. Colonies surrounded by a green zone C. Colonies without a zone of hemolysis around them D. No colonies

D. Bacterial cell walls are made of peptidoglycan and are susceptible to degradation by lysozyme. Gram-positive strains (such as Strep. viridans) are particularly vulnerable since their cell wall is unprotected by membrane like the cell walls of Gram-negative strains. Thus the cell wall of the Strep. viridans will be damaged by the lysozyme, so that the bacteria will not survive and no colonies will form (choice D is correct). Note that the hemolysis observed around some colonies is caused by the ability of bacteria to lyse red blood cells and is not related to lysozyme activity (choices A, B, and C are wrong).

Respiratory rate is regulated by many factors. Which of the following conditions is most likely to cause a decrease in breathing rate? A. Metabolic acidosis B. Low O2 concentration in the blood C. High CO2 levels in the blood D. High plasma pH

D. Consider following equilibria: CO2 + H2O --> H2CO3 --> HCO3-. In the blood, CO2 from the tissues is quickly converted to carbonic acid, which then dissociates into hydrogen ions (lowering the pH) and bicarbonate ions. Excess CO2 thus promotes metabolic acidosis, which is quickly compensated for by increasing the respiratory rate. This helps to eliminate CO2 from the blood and restore pH to normal (choices A and C are wrong). Low O2 concentrations in the blood would lead to an increase in respiratory rate (choice B is wrong); furthermore, blood O2 levels affect the respiratory rate only in situations where O2 availability is low, such as at high altitudes (above 7000 feet). Of the choices given, the most likely condition to induce a decrease in respiratory rate is high plasma pH (choice D is correct) since a reduction in respiratory rate would lead to an accumulation of CO2 in the blood, and a corresponding increase in H+.

Fe(CO)6Cl dissociates into two species, both charged, when dissolved in water. Which is the most likely expression for the dissociation constant (Ksp)? A. Ksp = [Fe+][CO]6[Cl−] B. Ksp = [Fe+][Cl−] C. Ksp = [Fe(CO)3+][Cl(CO)3−] D. Ksp = [Fe(CO)6+][Cl−]

D. Fe(CO)6Cl is a complex salt. Since CO is a neutral ligand, dissociation of one CO will not result in two ionic particles. Therefore, the dissociation must be Fe(CO)6Cl → Fe(CO)6+ + Cl−. This dissociation is represented well by choice D. Choice A is the equilibrium constant for total dissociation into eight free particles. Choice B can be eliminated as there will be no free Fe+ in solution if only two charged species form. Choice C can be eliminated since ligands like CO coordinate by donation of electron pairs, which anions such as Cl− are incapable of accepting.

Which of the following represents the correct sequence for embryogenesis? A. Fertilization → gastrulation → blastulation → neural tube formation → somite formation B. Fertilization → gastrulation → blastulation → somite formation → neural tube formation C. Fertilization → blastulation → neural tube formation → gastrulation → somite formation D. Fertilization → blastulation → gastrulation → neural tube formation → somite formation

D. Fertilization is the first step, followed by a series of rapid cell cleavages to form a hollow ball of cells called the blastulam (eliminate choices A and B). Next comes the gastrula, in which cells move into the interior of embryo to form the three germ layers (eliminate choice C). Gastrulation is followed by the formation of the neural tube, which will form the nervous system, followed by the formation of other organs and tissues, such as the somites that will differentiate into bones and muscle. This makes choice D the only possible correct order of events.

Vinca alkaloids are a class of anti-cancer drugs derived from the periwinkle plant. Once absorbed into a cell, they interfere with the polymerization of microtubules. These drugs can prevent cancer from spreading by disrupting: A. pseudopod formation, thereby preventing cellular locomotion. B. prophase, thereby halting mitosis. C. transcription, thereby halting production of crucial cell proteins. D. metaphase, thereby halting tumor growth.

D. Formation of microtubules is crucial for formation of the metaphase plate. Microtubules polymerize from the centrioles outward. They contact centromeres (to become kinetochore fibers) and "push" the chromosomes towards the center of the cell to form the metaphase plate. Without proper microtubule polymerization, metaphase (and the rest of mitosis) cannot occur. If mitosis cannot occur, then neither can tumor growth. Note that microtubules are also required during prophase in forming the mitotic spindle; however, some parts of prophase can still occur in the absence of microtubule formation (DNA condensation, loss of the nuclear membrane). This makes choice D a better choice than choice B, since virtually all of metaphase depends on proper microtubule polymerization. Pseudopod formation requires the growth of microfilaments (actin fibers), not microtubules (choice A is wrong), and transcription (RNA polymerization) does not require microtubules at all (choice C is wrong).

2,4-dinitrophenol (2,4-DNP) is a highly toxic substance which was sold to the public as a weight loss drug in the 1930s. It acts by permeabilizing the inner mitochondrial membrane (IMM) to ions. Which of the following is true of 2,4-DNP's effects on oxidative phosphorylation? A. It causes decreased flux of electrons through ATP synthase. B. It likely leads to a decrease in body temperature. C. It leads to decreased consumption of FADH2 and NADH by the electron transport chain proteins. D. It causes a decrease in the electromotive potential built up by the electron transport chain.

D. If the inner mitochondrial membrane became permeable to ions, then hydrogen ions would not need to go through the ATP synthase in order to re-enter the matrix. This would dissipate the proton gradient established by the electron transport chain, and decrease its potential to generate ATP (choice D is correct). FADH2 and NADH would still be consumed by the transport chain proteins, since the electron transport chain is not shut down; it's just that the proton gradient would be more easily dissipated (choice C is wrong). The dissipation of the gradient would result in heat production; note that this is similar to what happens in brown fat, used by hibernating animals to stay warm (choice B is wrong). Electrons do not flow through the ATP synthase (choice A is wrong).

Which of the following events takes place during Phase 0 of cardiac Action potential? A. A decrease in intracellular Ca2+ B. An efflux of intracellular K+ C. Closure of voltage-gated channels D. An influx of extracellular Na+

D. In both cardiac and skeletal muscle, as in neurons, the initial depolarization (Phase 0) is caused by the opening of voltage-gated sodium channels. When the channels open, sodium flows into the cell down the gradient maintained as part of the resting membrane potential (choice D is correct). Calcium and potassium flux do not occur in this period, and voltage-gated channels are opening, not closing (choices A, B, and C are wrong).

In addition to being used to produce alanine, pyruvate can be used to form: Glucose Acetyl-coA Fatty acids A. I only B. II only C. II and III only D. I, II, and III

D. Item I is correct: pyruvate is a substrate for gluconeogenesis, and can be used to form glucose. This eliminates choices B and C, which do not include glucose. Item II is also correct: pyruvate can be used to make acetyl-coA using the pyruvate dehydrogenase complex (the process between glycolysis and Krebs). This eliminates choice A, which does not include Item II. Item III is also correct: since pyruvate can be converted to acetyl-coA, it can be used to form fatty acids (which are made from acetyl-coA). Thus, choice D is correct.

Thirteen amino acids, including methionine, valine and proline, are glucogenic in humans. This means their α-keto acid carbon skeleton is converted to pyruvate during amino acid catabolism. After deamination, valine can therefore: I. Be converted into CO2 and H2O to generate ATP. II. Generate at least three NADH and two FADH2. III. Enter gluconeogenesis to generate glucose. A. I only B. III only C. I and II only D. I and III only

D. Item I is true: Amino acids are catabolized via deamination into α-keto acids and ammonia. Based on the information in the question stem, the α-keto acid formed from valine will be converted to pyruvate. Pyruvate can keep going through cellular respiration to generate CO2, H2O and ATP. Eliminate choice B. Item II is false: Pyruvate is converted into one acetyl-CoA (during which 1 NADH is made), and the acetyl-CoA would then generate three NADH and only one FADH2 as it cycles through the Krebs cycle. Eliminate choice C. Item III is true: Pyruvate can also enter gluconeogenesis to generate glucose. Eliminate choice A and choice D is correct.

Which of the following statements is/are true regarding reproductive health in society? I. Community clinics have a manifest function of providing women's health resources to lower-income citizens. II. A teenager who becomes pregnant is more likely to have children who also end up having babies at a very young age; this is an example of social reproduction. III. Obtaining contraception and pregnancy prevention information is an issue of both accessibility and availability in most lower-income communities. A. I and II only B. I and III only C. II and III only D. I, II, and III

D. Item I is true: a manifest function of a social institution it the intended purpose of that institution; providing women's health resources to lower-income citizens is the manifest function of community clinics, many of which provide free counseling and resources (choice C can be eliminated). Item II is true: social reproduction refers to the structures in society that transmit social inequality from one generation to the next Item. In other words, the lack of knowledge and access to resources which might have made it more likely for a teenager to become pregnant in the first place will also make it more difficult for that teenager to provide a better life to her children, which tends to perpetuate a cycle of poverty and makes it harder for the next generation to rise above their circumstances. Therefore, the fact that women who get pregnant at an early age are more likely to raise children who also end up having babies very young is an example of social reproduction (choice B can be eliminated). Item III is true: most lower-income neighborhoods do not have as many healthcare resources (both contraception and information) available to them. For example, fewer doctors tend to practice in poorer communities, schools may not have the resources to provide sexual education, and community clinics may not have the fund for outreach or free condoms. These are all issues of availability (the needed resources are not available). Similarly, even when resources do exist, there are also issues of accessibility that also prevent individuals in lower-income communities from obtaining the information or resources they need. For example, even if a low-cost clinic is in the neighborhood, it might be so overbooked that it is difficult to get an appointment when needed. Therefore, obtaining contraception and pregnancy prevention information is an issue of both accessibility and availability in most lower-income communities (choice A can be eliminated; choice D is correct).

All of the following biological developmental milestones that change the brain occur during the adolescent years EXCEPT: A. Cell proliferation (particularly in the limbic system and prefrontal lobes) B. Myelination C. Synaptic pruning (of unused or unnecessary connections) D. Codification of neural networks for basic motor skills

D. Neural network development for motor skills is part of early brain development and is essentially complete for basic motor skills (such as crawling, walking, jumping, and writing) before adolescence (choice D does not occur during adolescence and is therefore correct). Cell proliferation, particularly in the prefrontal lobes and in the limbic system, is one of the changes to the brain that occurs during adolescence, as the prefrontal cortex is required for abstract thinking (choice A is wrong). Myelination, the creation of myelin sheaths around neurons in the brain to strengthen connections, is another change that occurs during adolescence (choice B is wrong). Synaptic pruning, which clears the brain of unnecessary connections, is the third major developmental change in the brain during adolescence (choice C is wrong).

Which of the following is NOT a cluster B personality disorder? A. Antisocial personality disorder B. Borderline personality disorder C. Histrionic personality disorder D. Paranoid personality disorder

D. Personality disorders are grouped based on similarities in their characteristics. Cluster B disorders are characterized by heightened emotional arousal, as well as by dramatic, erratic, or impulsive behavior. Paranoid personality disorder is characterized essentially by distrustful behavior; it is a cluster A (odd) disorder and not a cluster B (dramatic) disorder (choice D is not a cluster B disorder and is therefore correct). Antisocial, borderline, and histrionic personality disorders are all characterized by poor emotional regulation and are cluster B disorders (choices A, B, and C are wrong).

Complete reduction of acetic acid will produce which of the following? A. O2(g) B. CO2(g) C. CH3CHO D. CH3CH2OH

D. Reduction of acetic acid will initially produce the aldehyde given as choice C, where the carbonyl carbon has been reduced from an oxidation state of +3 to +2. Choice D is the result of further reduction to an oxidation state of +1, while choices A and B do not correspond to the reduction of acetic acid.

If a bisexual female reports feeling social pressure to be either lesbian or straight, which causes her significant stress, what term best describes the stress this subject is feeling about her social role? A. Role conflict B. Master status C. Role exit D. Role strain

D. Role strain is when having a single status results in conflicting expectations; if a bisexual female reports feeling social pressure to be either lesbian or straight, which causes her significant stress, she is experiencing conflicting expectations about her bisexual identity (choice D is correct). Role conflict happens when there are conflicting societal expectations for multiple statuses held by the same person. For example, a male kindergarten teacher—societal expectations for being a man and being a kindergarten teacher easily come into conflict (choice A is wrong). Master status is when one part of someone's identity comes to dominate their identity, to the exclusion of other parts of their identity (choice B is wrong). Role exit is the process of disengaging from one role in order to take up another. An example is the process of disengaging from a "pre-med" identity to take on a "medical student" identity (choice C is wrong).

The myelin sheath is a dielectric that surrounds axonal membranes and facilitates electrical transmissions of neuronal signals. Demyelination is implicated in the exacerbation of Alzheimer's symptoms. Impairment of cognitive abilities due to demyelination most likely occurs because: A. increased speed of processing results in improved episodic memory. B. decreased speed of processing results in improved semantic memory. C. decreased speed of processing results in improved episodic memory. D. decreased speed of processing results in impaired semantic memory.

D. Semantic memory refers to the memory of meanings and other concept-based knowledge. As demyelination occurs, neuronal signals that carry meanings slow down, damaging the memory of them (choice D is correct). Episodic memory is the ability to contextualize the memory of autobiographical events; increased speed of neuronal processing could result in improved episodic memory, but this would not impair cognitive abilities and is not what the question stem asks (choice A is wrong). Decreased speed of neuronal processing would not result in memory improvement, whether semantic or episodic (choices B and C are wrong).

Which of the following is NOT a theory that accounts for gender differences? A. Gender stereotyping B. Gender role preference C. Gender schema theory D. Sex drive

D. Sex drive in humans is a function of physiological (e.g., hormonal) and non-physiological factors, but has nothing to do with gender differences (choice D is not a theory that accounts for gender differences and is therefore correct). Gender stereotyping describes the all-pervasive ideas in a culture to the effect that certain types of clothes, comportment, toys, colors, etc. are appropriate for only one gender (choice A accounts for gender differences and is therefore wrong). Gender role preference involves one's decision that, despite having the physiological and biological characteristics of one gender, he or she identifies with the other gender and proceeds to comport him or her self accordingly (choice B accounts for gender differences and is therefore wrong). Gender schema theory explains how gender differences are transmitted; the theory incorporates gender stereotypes and biological characteristics (choice C accounts for gender differences and is therefore wrong)

A chemist prepares a 1 M solution of sulfuric acid. Which of the following gives the relative concentrations of the species in solution? A. [H2SO4] > [HSO4-] > [SO42-] B. [SO42-] > [HSO4-] > [H2SO4] C. [HSO4-] > [H2SO4] > [SO42-] D. [HSO4-] > [SO42-] > [H2SO4

D. Since sulfuric acid is a strong acid and dissociation of the first proton is complete, undissociated H2SO4 will have the lowest relative concentration, eliminating choices A and C. Since the dissociation constant for the loss of the second proton is well below 1 (~ 10-2), HSO4- will exist in greater concentration than SO42-, making choice D the correct answer.

The hippocampus is a subpart of which brain structure? A. Executive functioning B. Basal ganglia C. Occipital lobe D. Limbic system

D. The hippocampus is part of the limbic system of the brain; a system which is responsible for many aspects of human functioning, including emotions and memory (choice D is correct). Executive functioning is generally thought to be regulated by the frontal lobes, which control inhibition and attention (choice A is wrong). The basal ganglia is situated at the base of the forebrain and is mainly comprised of striatum, the globus pallidus, the substantia nigra, the nucleus accumbens, and the subthalamic nucleus. It is associated with voluntary movement, procedural learning, and emotions, but is not associated with episodic memory as the hippocampus is (choice B is wrong). The occipital lobe of the brain is at the back of the head, and is primarily associated with vision (choice C is wrong).

At approximately what frequency does a protein with a purely beta sheet conformation exhibit a minimum in its CD spectrum? Passage states that beta sheet proteins have CD spectral minima at 218 nm A. 7.27 × 10−16 Hz B. 1.38 × 10−3 Hz C. 7.27 × 102 Hz D. 1.38 × 1015 Hz

D. The passage states that beta sheet proteins have CD spectral minima at 218 nm. The wavelength and frequency of light are related through the equation c = λf. Solving for frequency, we get f = c / λ = (3 × 108 m/s) / (218 × 10−9 m) = 1.38 × 1015 Hz, which corresponds to choice D. Choices A, B, and C can be incorrectly obtained if the exponent used for nm is interpreted as positive instead of negative and/or the equation for frequency is flipped.

Antibiotics possessing a β-lactam, such as penicillin, will irreversibly bind and inactivate TPases. Which one of the following amino acid(s) is LEAST likely to be in the active site of a TPase? Serine Lysine Valine Alanine A. I only Your Answer B. IV only C. I and II only D. III and IV only Correct Answer

D. The passage states that penicillin will irreversibly bind to TPase and stop its enzymatic activity. Penicillin possesses a β-lactam functional group, or a 4-membered lactam (cyclic amide). This highly strained ring is susceptible to ring-opening in the presence of nucleophiles. Therefore, the TPase enzyme must have a nucleophilic amino acid in its active site that can react with a β-lactam. Roman numerals I and II are both nucleophilic, possessing an alcohol and amine side chain, respectively (eliminate choices A and C). Roman numerals III and IV possess non-nucleophilic hydrocarbon side chains, isopropyl and methyl, respectively. Thus, choice D is the answer because it contains both of these correct options.

Why is the copper(I) catalyst not necessary to link the fluorophore bearing the cyclooctyne to the stem peptide, even though it is required for other alkynes? A. The copper would increase the Ea. B. The reaction is spontaneous. Your Answer C. The Ea of the reaction is low due to the high reactivity of the azide. D. The Ea of the reaction is low due to the ring strain from the cyclooctyne

D. The role of any catalyst is to lower the activation energy (Ea) of a reaction (eliminate choice A), thereby increasing the reaction rate. Catalysts cannot, however, force a nonspontaneous reaction to occur, therefore both the reactions that use and do not use the catalyst must be spontaneous (eliminate choice B). The passage indicates that the copper catalyst is needed to link the azide to the non-activated alkyne. This implies that the azide is a stable functional group and therefore unreactive (eliminate choice C). The activation energy of a reaction can be lowered by stabilizing the transition state of the rate-determining step, or by increasing the relative energy of a substrate that is involved in the kinetics of the reaction (less stable molecules are more reactive). The cyclooctyne ring is highly strained due to the linear geometry of the alkyne. Therefore this substrate is highly reactive and will readily react with an azide without the need for a rate increase from the copper(I) catalyst (choice D is correct).

Because researchers chose to use a questionnaire, which of the following was missing from this study? A. Dependent variables B. Scientific method C. Independent variables D. A control group

D. The use of a questionnaire means that the researchers did not utilize a control group and thus did not have two groups participating simultaneously under different circumstances; this would simply not be possible with this methodology (choice D is correct). The dependent variables in this study are the preferences regarding age and gender of the students polled in the questionnaire—what the researchers were looking to study (choice A is wrong). The scientific method, in which hypotheses are empirically tested through the careful and systematic collection of relevant data. was used in this study and is ideally used in all scientific research (choice B is wrong). The independent variable here is the group of students polled (those between the ages of 18 and 30, identifying as either male or female; choice C is wrong).

Which of the following hormones acts on target cells via a second-messenger system? A. Thyroid hormone B. Aldosterone C. Cortisol D. Epinephrine

D. Thyroid hormone, aldosterone, and cortisol are all small hydrophobic molecules that passively diffuse through the plasma membrane to bind to receptors inside the cell in the cytoplasm and nucleus. These receptors then regulate transcription, without the use of second-messenger systems (choices A, B, and C are wrong). A hydrophilic hormone like epinephrine, however, binds to cell-surface receptors and activates adenylate cyclase to make cAMP, a second messenger (choice D is correct). Note that in general, steroid hormones exert their effects by modifying transcription, while protein hormones utilize second-messenger systems. Thyroid hormone, a protein hormone, is an exception.

Wernicke's area would be associated primarily with which of the following activities? A. Decoding the text B. Circling the ratings on the IITQ C. Asking a research assistant for help D. Comprehending the text

D. Wernicke's area is a region in the left hemisphere of the brain responsible for language comprehension (choice D is correct). It is not responsible for decoding, which involves pronouncing the sounds of letters and words (choice A is wrong), nor is it associated with motor activity (choice B is wrong). Broca's area, not Wernicke's area, is associated with language production (choice C is wrong).

The ΔH of the formation of ammonia is given as -91.8 kJ/mol. Information provided in the passage indicates that the decomposition of ammonia gas is an: A. exothermic reaction with a negative ΔS°. B. exothermic reaction with a positive ΔS°. C. endothermic reaction with a negative ΔS°. D. endothermic reaction with a positive ΔS°

The ΔH of the formation of ammonia is given as -91.8 kJ/mol. Therefore, the decomposition of ammonia must have a ΔH of +91.8 kJ/mol, and must be endothermic. Note that the decomposition of ammonia produces 4 molecules (1 nitrogen gas molecule and 3 hydrogen gas molecules) for every two molecules of ammonia, so the change in entropy (ΔS) of the decomposition is positive. This agrees with the decrease in yield of ammonia at high temperatures, where the positive entropy term would favor dissociation. Since the formation of ammonia is said to be negligible under the quoted conditions,ΔG° for dissociation must be negative.

Insulin function

acts to lower blood glucose and will not be secreted in response to low blood glucose

longitudinal study design

allow researchers to follow one group of participants over time

Pyrophosphatases function

are members of the acid hydrolase family of enzymes that cleave diphosphate bonds.

glycoside hydrolases degrade

carbohydrates

monoanime oxidases degrade

certain neurotransmitters such as norepinephrine and epinephrine

A phosphodiester bond is a

covalent bond formed between a phosphate group and a pentose sugar. This is the type of bond used to attach the phosphate backbone to nucleotides

Role conflict

happens when there are conflicting societal expectations for multiple statuses held by the same person. For example, a male kindergarten teacher—societal expectations for being a man and being a kindergarten teacher easily come into conflict

Cluster B disorders are characterized by

heightened emotional arousal, as well as by dramatic, erratic, or impulsive behavior.

a lower Km (analogous to KT) indicates a lower OR higher binding preference

higher binding preference

A looking-glass self describes

how people shape their self-concepts based on how they think others perceive them

Semantic memory

is memory of concepts and information

procedural memory

is memory of how to execute motor tasks

optimistic bias

is people's tendency to assess the probability of personal success in future endeavors with more optimism than is warranted by actual facts

Glucagon function

is secreted by alpha cells in the pancreas and acts to elevate plasma glucose if it falls too low

somatostatin

is secreted in response to raised glucose or other nutrients and acts to repress digestive functions

adaptation effect

is the phenomenon by which an organism becomes accustomed to a sensory stimulus, which results in a higher threshold for detection of that stimulus.

Role exit

is the process of disengaging from one role in order to take up another. An example is the process of disengaging from a "pre-med" identity to take on a "medical student" identity

Role strain

is when having a single status results in conflicting expectations; for example if a bisexual female reports feeling social pressure to be either lesbian or straight

if an element has an Eº < 0, that means

its reduction is not spontaneous, making it a poor oxidizing agent

Lipases degrade

lipids

The role of any catalyst is to

lower the activation energy (Ea) of a reaction (eliminate choice A), thereby increasing the reaction rate. Catalysts cannot, however, force a non-spontaneous reaction to occur, therefore both the reactions that use and do not use the catalyst must be spontaneous

The levels of processing theory states that

people remember information better when they process it more deeply. For example, people who are made to answer questions about material they read will remember the material better than those who simply read it.

The representativeness heuristic

refers to a problem-solving strategy by which people make choices based on their perception of which option is most characteristic of the issue in question.

Episodic memory

refers to memory of one's personal experiences

The availability heuristic

refers to people's tendency to solve problems or estimate probabilities based on information that is most readily available, or accessible, to their minds.

negative reinforcement

refers to the removal of a negative (or undesirable) stimulus in response to the target behavior

Conservation (Piaget)

sed to describe children's recognition of constancy and consistency in physical matter despite changes in shape or container. Conservation is realized through three insights: identity, reversibility, and reciprocity. This awareness is characteristic of the stage of concrete operational thought, which is usually achieved between the ages of 6 or 7 and 11 or 12.

The activation energy of a reaction can be lowered by

stabilizing the transition state of the rate-determining step, or by increasing the relative energy of a substrate that is involved in the kinetics of the reaction (less stable molecules are more reactive)

Sapir-Whorf hypothesis,

suggests that the structure of a language determines/influences the thought and behavior characteristic of the culture in which it is spoken.

should the dilution of buffer solution affect the buffer solution's pH?

the dilution of the buffer solution should not affect the pH

positive reinforcer

the event that is perceived by the subject as desirable and which follows the target behavior

A specific phobia is defined as .

the fear of a specific object or situation These are divided into four categories based on the specific triggers: animal, medical, natural environmental, and situational phobias (e.g., arachnophobia is a common specific animal phobia of spiders)

would an increase in rotational inertia result in an increased or decreased tendency to rotate

the increase in rotational inertia will imply less tendency to rotate (that is, less tendency to topple off the cable), thus making balancing easier.

what happens when the net torque on an item/object is not zero?

the item/object would accelerate rotationally

Gestalt psychology emphasizes

the mental processes that perceive the "whole" of objects or situations. This school of psychology has been particularly influential in describing tendencies of human visual perception

the pH of a buffer solution depends on

the ratio of the concentrations of acid to base, and not their absolute quantities.

define hemostasis

the stoppage of blood flow

Km is defined as

the substrate concentration at which the velocity is half of Vmax.


Kaugnay na mga set ng pag-aaral

All of the following statements are correct regarding credit life insurance EXCEPT Benefits are paid to the borrower's beneficiary. The amount of insurance permissible is limited per borrower. Premiums are usually paid by the borrower. Benefit

View Set

Nomenclatura compuestos inorgánicos

View Set

Protection Under Consumer Credit Laws

View Set

Corporate Citizenship: Social Responsibility, Responsiveness, and Performance

View Set

Types of houses and other vocabulary on "house and home"

View Set

SAM 3250J Final Exam (new questions)

View Set